Đến nội dung

superpower

superpower

Đăng ký: 21-09-2015
Offline Đăng nhập: 07-01-2019 - 16:51
**---

#655748 Đề chọn đội tuyển Toán tỉnh Đăk Lăk năm 2016

Gửi bởi superpower trong 27-09-2016 - 17:18

\

Câu 2: Tìm số dương $k$ lớn nhất thỏa mãn $(a+b+c)(\frac{1}{a+b}+\frac{1}{b+c}+\frac{1}{c+a}-k)\geq k$, với $a,b,c$ là các số thực không đồng thời bằng 0 thỏa mãn $a+b+c=ab+bc+ca$.
 

Thay $c=0, a=b=2  => k \leq 1 $

Ta chứng minh $k =1 $ thỏa 

Khi $k=1$, ta cần chứng minh

$(a+b+c)(\frac{1}{a+b} + \frac{1}{a+c} + \frac{1}{b+c} -1 ) \geq 1 $

Quy đồng và chuyển pqr, ta cần chứng minh 

$p . ( \frac{p^2+p}{pq-r} -1) \geq 1 $ 

Mà do $r \geq 0 $

Nên ta chỉ cần chứng minh 

$p(\frac{p^2+p}{p^2} -1 ) \geq 1$

Mà thật ra đây chi là đẳng thức

Do đó ta có đpcm 




#654495 Đề thi chọn đội tuyển quốc gia tỉnh Quảng Bình

Gửi bởi superpower trong 17-09-2016 - 16:20

Bởi vì công thức tổng quát kia là khi n chạy và x cố định, không phải x chạy. Nói cách khác, $f(x)=x+c$  chỉ đúng với chính xác 1 giá trị x thôi.

Cụ thể hơn, với mỗi x sẽ tồn tại một số c khác nhau, không phải tổng quát cho tất cả x. Lý do: giá trị của c tính theo x1 và x2, mà x1 x2 không cố định theo n

Bằng chứng rõ ràng là hàm f(x) = -x cũng có f(f(x))=x nhưng rõ ràng f(x) không phải dạng x+c

 

 

Tớ không thấy từ "sai bét" làm sao cả tại vì tất cả đều dùng từ này hàng ngày với nghĩa bình thường, trừ việc đối với vài người nhạy cảm có thể thấy nó offended. Nếu ai thấy bị offended bởi 1 từ như thế thì tớ xin lỗi luôn. Tớ cũng không thể làm hài lòng tất cả mọi người được.

Thật ra người sai chính là bạn

Thật ra còn 1 công thức nữa là $f(x)=-x +c $ nhưng mình không tiện đưa ra ở đây vì nó không thỏa thôi

Bài trên mình thay $c_2$ bởi $c_1$

Nếu bạn thay $c_1$ bởi $c_2$ thì sẽ ra công thức $f(x)=-x+c $

Người không hiểu phương pháp chính là bạn

Nếu chúng ta có 1 mối quan hệ giữa $f(f(x)), f(x), x $ với ĐK là các hàm số xét trên tập các số dương hoặc nhỏ hơn thì hoàn toàn có thể dùng pp sai phân

Bạn học lại đi

Bạn có thể xem lại cuốn PTH của thầy Nguyễn Tài Chung phần ứng dụng dãy số để giải PTH

Hoặc là trong kỉ yếu Gặp Gỡ Toán Học năm nay có bài viết của anh Võ Quốc Bá Cẩn về phần này

Bạn có thể xem bài toán 6 trang 83 trong quyển kỉ yếu này

Thân ái




#654352 Đề thi chọn đội tuyển quốc gia tỉnh Quảng Bình

Gửi bởi superpower trong 16-09-2016 - 00:29

Câu 7 : Tìm tất cả hàm số $f : N^{*} \to N^{*}$ sao cho ba số $a,f(b),f(b+f(a)-1)$ luôn là độ dài ba cạnh của một tam giác với mọi $a,b \in N^{*}$ 

Thay $a=b=1 => 1, f(1), f(f(1)) $ là 3 cạnh của tam giác

Do đó  $f(1) + 1 \geq f(f(1)) +1 => f(1) \geq f(f(1))$

Mặt khác $a+f(b+f(a)-1) \geq f(b) +1 => f(f(1)) \geq f(1)$

Do đó $f(1)=f(f(1)) $ 

Ta có $a+f(b) \geq f(b+f(a)-1) +1 => f(b) \geq f(b+f(1)-1) $

Ta cũng có $a+f(b+f(a)-1) \geq f(b)+1 => f(b+f(1)-1) \geq f(b) $

Do đó $f(b)= f(b+f(1)-1) $

TH1: $f(1) \neq 1 =>f$ tuần hoàn

Mà do $f$ từ $N^{*}-> N^{*} => f $ bị chặn

Khi đó sẽ tồn tại $M=max f(x) $

Mà ta dễ dàng có $f(f(a)) +f(1) \geq a+ 1$

Cho $a -> +\infty $ ra vô lí

Do đó $f(1)=1$ 

$a+f(b) \geq f(b+f(a)-1) +1$

Thay $b=1 => a+1 \geq f(f(a)) +1 => a \geq f(f(a)) $

Mặt khác $f(b+f(a)-1) +f(b) \geq a+1 $

Cho $b=1 => f(a) +1 \geq a+1 => f(f(a)) \geq a $

Do đó $f(f(a))=a$ 

Do đó, xét dãy $x_n=f(f(...(x)...)) $ ( $n$ lần $f$ )

Khi đó ta có $x_0=x; x_1=f(x) $

Và $x_{n+2} = x_n $

Xét PTĐT có 2 nghiệm phân biệt là $x=1;-1$

Do đó nghiệm tổng quát sẽ là 

$x_n=c_1+ c_2(-1)^n $

với $c_1 + c_2 = x $

      $c_1 - c_2 = f(x) $

Do đó $c_1= x- c_2 $

Thay vào , ta đc $x_1 = f(x) =x - 2c_2 $

Do đó $f(x)= x+c $

Thử lại , ta nhận nghiệm $f(x)=x$  

Thử lại thỏa




#654251 có bao nhiêu bộ nghiệm

Gửi bởi superpower trong 15-09-2016 - 10:17

cho phương trình $x_{1}+x_{2}+x_{3}+x_{4}+x_{5}+x_{6}+x_{7}+x_{8}+x_{9}=2009$ tìm số các bộ nghiệm (x1,x2,x3....x9) của pt sao cho các các nghiệm x1,x2,x3....x9 đều lẻ

Đặt $x_1=2x_1' +1 $

Khi đó $x_1' không âm $

Do đó, ta đưa về phương trình

$x_1' + ... + x_9'=1000 $

Với $x_1',...,x_9' $ không âm

Đây là bài toán chia kẹo Euler




#654099 Đề thi chọn đội tuyển Nguyễn Du (Đăk Lăk)-Vòng 2

Gửi bởi superpower trong 14-09-2016 - 08:25

 

ĐỀ THI CHỌN ĐỘI TUYỂN LẦN 2

NĂM HỌC 2016-2017

 

 

Bài 2(6 điểm):

 

$2)$ Tìm tất cả hàm số $f:\mathbb{R}\rightarrow \mathbb{R}$ thỏa mãn:

$f(x^3)+f(y^3)=(x^2-xy+y^2)\left ( f(x)+f(y) \right )\ ;\forall x,y\in \mathbb{R}$

 

Câu dãy hình như sai đề nha anh

câu hàm 

$x=y=0 => 2f(0)=0 => f(0)=0 $

$y=0 => f(x^3) = x^2f(x) $ 

Do đó $f(x^3) + f(y^3) = x^2f(x) + x^2f(y) -xyf(x) -xyf(y) +y^2f(x) +y^2f(y ) $

Do đó $x^2f(y) + y^2f(x) = xy( f(x) +f(y) )<=>x(xf(y) -yf(x) ) + y(yf(x)-xf(y)) =0 => (xf(y)-yf(x))(x-y) =0  $ 

Khi $x=y => xf(y)=yf(x) $

Mà khi $x \neq y => xf(y)=yf(x) $

Do đó $xf(y)=yf(x) , \forall x,y \in R $

Do đó thay $y=1 => f(x) =xf(1) $

Thử lại $f(x)=x$ 




#653626 CMR: $\frac{1}{a+b}+\frac{1}...

Gửi bởi superpower trong 10-09-2016 - 21:13

Tại sao đoạn Cauchy_Schwwarz lại ra như thế kia vậy a

E nhớ là BĐT Cauchy-schwwarz đâu có như vậy

Với cả dấu bằng của a, em nghĩ có vấn đề, áp dụng C-S ==> a=b=c ==> ko t/m

Đó là C-S dạng engel đó bạn

$\sum \frac{c}{a+b} = \sum \frac{c^2}{ca+cb} \geq \frac{(a+b+c)^2}{2(ab+bc+ca)} $

Về chỗ dấu bằng

Thì theo bđt C-S thì xảy ra khi các biến tỉ lệ với quy ước mẫu bằng 0 thì tử bằng 0

Do đó vẫn đảm bảo dấu bằng tại biên của bài toán




#653612 $A=\sum x^{3} +8(xy^{2}+yz^{2}+zx^...

Gửi bởi superpower trong 10-09-2016 - 19:29

Cho $x,y,z\geq 0;x+y+z=4.$

Tìm max: $A=\sum x^{3} +8(xy^{2}+yz^{2}+zx^{2})$

Bài này mình mò được điểm rơi là $\left ( 0;2;2 \right )$ mà chưa tìm được cách.

Đặt $f(x,y,z) = \sum x^3 + 8(xy^2+yz^2+zx^2) $

Ta có $f(x+z,y+z,0) = (x+z)^3 + (y+z)^3 + 8(x+z)(y+z)^2 $

Ta chứng minh $f(x,y,z) \leq f(x+z,y+z,0 ) $

Là hiển nhiên

Do đó, ta càn tìm max của

$B= a^3+b^3+8.ab^2 $ với $a+b \leq 4 $

Tới đây dễ rồi




#653256 $I$ là tâm đường tròn nội tiếp tam giác $ABE$.

Gửi bởi superpower trong 07-09-2016 - 22:59

Bài toán: Cho tam giác $ABC$ cân tại $A$ có $H,M$ lần lượt là trung điểm của $BC,AC$. Đường tròn ngoại tiếp tam giác $BCM$ cắt đoạn $AH$ tại $D$ và đường tròn ngoại tiếp tam giác $ABD$ cắt đoạn $BM$ tại $K$. Gọi $I$ là giao điểm của $AK$ và $BD$, $E$ là giao điểm của $CI$ với $BM$. Chứng minh:
1.Tam giác $AKC$ vuông.
2.$I$ là tâm đường tròn nội tiếp tam giác $ABE$.

Bài hình đầu tiên mình giải trên VMF :))

Ta chứng minh  $\triangle AKC $ vuông như sau

$\widehat {KAM} =\widehat {BHD}=\dfrac{1}{2} \widehat{BDC} = \dfrac{1}{2} \widehat{BMC}  $

Do đó $MK=MA $

Suy ra $\triangle AKC$ vuông

b/  Ta có $IK.ID=ID.IB => I $ thuộc trục đẳng phương của $(BMC)$ và $(AC) $

Do đó $CI $ cắt $(BMC)$ tại $J $

Mặt khác, ta cũng có $\widehat {MBC} = \widehat{MJC} = \widehat{MCJ} $

Do đó $MA^2=MC^2=ME.MB $

Từ đó biến đổi góc thêm 1 tí là ra

Hình gửi kèm

  • Zalo_ScreenShot_7_9_2016_225214.png



#652935 tìm tất cả các stn n sao cho n chia hết cho [$\sqrt{n}...

Gửi bởi superpower trong 05-09-2016 - 20:20

tìm tất cả các stn n sao cho n chia hết cho [$\sqrt{n}$]

Đặt $[\sqrt{n} ] = d $

Khi đó $d \leq \sqrt{n} < d+1 $

$d^2 \leq n < d^2+ 2d $

Do đó $n= d^2 + r$

Với $r= 1,2,...,2d-1 $

Mặt khác $n \vdots d => r \vdots d $

Do đó, $r=d $

Vậy $n= d^2+ d $

Thử lại thỏa 




#652777 Đề chọn đội tuyển Quốc Gia môn Toán

Gửi bởi superpower trong 04-09-2016 - 17:28

 

Bài toán 3: Cho $P(n)$ là một đa thức( hệ số thực) của biến tự nhiên $n$ thỏa mãn:

$P(n)=1^{2003}+2^{2003}+...+n^{2003},\forall n\in N^{*}$.

Chứng minh rằng: đa thức $P(n)$ chia hết cho đa thức $Q(n)=n^2(n+1)^2$

 

 

Bài 3 có sai gì không bạn

Dễ thấy $P(1) = 1$ không chia hết cho $Q(1) $

$P(2) $ cũng vậy 




#652776 $x_{n+1} = \dfrac{x_n}{2} + \dfrac{n^2}{4n^2+a}.\sqrt{x_n...

Gửi bởi superpower trong 04-09-2016 - 17:25

Full cho em hộ anh. Em mới tìm hiểu phần này thôi =))

Thật ra bài này em có thể đọc trong phần lời giải và bình luận kì thi VMO 2015 của thầy Dũng

Có 2 cách giải

Anh có 1 lời giải khác, dùng bổ đề, nhưng chứng minh tương tự thôi nên không đăng

File gửi kèm




#651983 CMR: $\sum\frac{a}{b+c}+\frac{ab...

Gửi bởi superpower trong 30-08-2016 - 18:13

Ngay từ đầu bài toán em chuẩn hóa luôn a+b+c=3 thì có đc ko a?

đc em. Những sẽ không quy đc về soS




#651688 CMR: $\sum\frac{a}{b+c}+\frac{ab...

Gửi bởi superpower trong 28-08-2016 - 19:54

Anh có thể giúp em viết rõ phần màu xanh này đc ko ạ, em thấy hơi khó hiểu ~O)  

2a+b+c > a+b+c vì a,b,c >0, mới cả anh viết hộ em dấu bằng xảy ra khi nào ạ?

 

BĐT $<=> \sum \frac{a}{b+c} -\frac{3}{2} + \frac{abc}{2(a^3+b^3+c^3} - \frac{1}{6} \geq 0 $

        $<=> \sum (a-b)^2 . \frac{1}{2(a+c)(b+c)} - \sum (a-b)^2. \frac{a+b+c}{12(a^3+b^3+c^3) } \geq 0 $

        $<=> \sum (a-b)^2 . (\frac{1}{2(a+c)(b+c)} -\frac{a+b+c}{12(a^3+b^3+c^3) } ) \geq 0 $

Giả sử $a \geq b \geq c $

Suy ra đc $S_c \leq S_b \leq S_a $

Do đó, ta cần chứng minh 

$S_b + S_c >0 <=> 3(2a+b+c)(a^3+b^3+c^3) \geq (a+b+c)(a+b)(b+c)(c+a) $

Do tính thuần nhất, ta có thể chuẩn hóa $a+b+c =3 $

Và chú ý rằng $2a+b+c \geq a+b+c $

Chuyển $pqr $

Do đó, ta chỉ cần chứng minh $3(p^3-3pq+3r) \geq pq -r $

$<=> r \geq \frac{30q-81}{10} $

Nếu $q \leq \frac{27}{10} $ ta có đpcm 

Nếu $3 \geq q \geq \frac{27}{10} $, áp dúng bdt Schur bậc 3, ta cần chứng minh 

$\frac{12q-27}{9} \geq \frac{30q-81}{10} <=> 150q \leq 459$ đúng

Vậy ta có đpcm 

Bây giờ, đẳng thức xảy ra khi $a=b=c$

Vì bên ngoài các đại lượng $S_a, S_b , S_c $ đã có $(a-b)^2, (b-c)^2, (c-a)^2 $ rồi

Mặt khác, theo tiêu chuẩn chứng minh SOS cho $a \geq b \geq c $

Là $S_a(b-c)^2 + S_b(a-c)^2  +S_c( a-b)^2 \geq 0 $

Nếu có $S_b \geq 0 , S_b + S_c \geq 0 , S_b+ S_a \geq 0 $ là ta có dương

Cái trên anh áp dụng luôn

Do có $S_c \leq S_b \leq S_a $

nên chỉ cần chứng minh $S_b + S_c >0 $ thôi

Khi đó anh sẽ thử chứng minh 

$3(2a+b+c)(a^3+b^3+c^3) \geq (a+b+c)(a+b)(b+c)(c+a) $

Mà đã có $2a+b+c \geq a+b+c $

Nên chỉ cần chứng minh $3(a^3+b^3+c^3) \geq (a+b)(b+c)(c+a) $ thôi

Mà cái này anh quy về $pqr$ để chứng minh thì thấy nó đúng




#651386 Chứng minh rằng: $a+b+c \leq 1+abc$

Gửi bởi superpower trong 26-08-2016 - 20:30

Cho $a,b,c$ là các số thực dương thỏa mãn $(a+1)(b+1)(c+1)=1+4abc$

Chứng minh rằng: $a+b+c \leq 1+abc$

Thay $(a;b;c) \rightarrow (\dfrac{1}{a};\dfrac{1}{b};\dfrac{1}{c})$, ta được bài toán mới

Ta quy về pqr, bđt cần chứng minh trở thành

Cho $p+q=3$

a/ Chứng minh $q < \ 1 +r $

Ta cần chứng minh $r > 2 -p $

Theo bđt Schur bậc 3, ta $p^3 -4pq +9r \geq 0 <=> r \geq \dfrac{12p-4p^2-p^3}{9} $

Ta cần chứng minh $\dfrac{12p-4p^2-p^3}{9} \geq 2-p <=> p^3+4p^2-21p+18 <0<=> 1,24 <p<2 $

Vậy bđt đúng khi $1,24<p<2 $

Mặt khác, ta $3=p+q \leq p+\dfrac{p^2}{3} <=> p \geq 1,8 $

Nếu $p \geq 2 $thì $2-p \leq 0 \leq r $

Do đó bài toán được chứng minh




#651380 CMR: $\sum\frac{a}{b+c}+\frac{ab...

Gửi bởi superpower trong 26-08-2016 - 20:07

Cho a,b,c > 0. CMR:

$\frac{a}{b+c}+\frac{b}{a+c}+\frac{c}{a+b}+\frac{abc}{2(a^3+b^3+c^3)} \geq \frac{5}{3}$

BĐT $<=> \sum \frac{a}{b+c} -\frac{3}{2} + \frac{abc}{2(a^3+b^3+c^3} - \frac{1}{6} \geq 0 $

        $<=> \sum (a-b)^2 . \frac{1}{2(a+c)(b+c)} - \sum (a-b)^2. \frac{a+b+c}{12(a^3+b^3+c^3) } \geq 0 $

        $<=> \sum (a-b)^2 . (\frac{1}{2(a+c)(b+c)} -\frac{a+b+c}{12(a^3+b^3+c^3) } ) \geq 0 $

Giả sử $a \geq b \geq c $

Suy ra đc $S_c \leq S_b \leq S_a $

Do đó, ta cần chứng minh 

$S_b + S_c >0 <=> 3(2a+b+c)(a^3+b^3+c^3) \geq (a+b+c)(a+b)(b+c)(c+a) $

Do tính thuần nhất, ta có thể chuẩn hóa $a+b+c =3 $

Và chú ý rằng $2a+b+c \geq a+b+c $

Chuyển $pqr $

Do đó, ta chỉ cần chứng minh $3(p^3-3pq+3r) \geq pq -r $

$<=> r \geq \frac{30q-81}{10} $

Nếu $q \leq \frac{27}{10} $ ta có đpcm 

Nếu $3 \geq q \geq \frac{27}{10} $, áp dúng bdt Schur bậc 3, ta cần chứng minh 

$\frac{12q-27}{9} \geq \frac{30q-81}{10} <=> 150q \leq 459$ đúng

Vậy ta có đpcm